How to solve this nonlinear recurrence relations?












1












$begingroup$


I know how to solve linear homogeneous and non-homogeneous recurrence relations. But today I found this problem.



$$F_1=1$$
$$F_2=7$$
$$F_3=13$$
$$F_n = 3F_{n-1} + k(F_{n-2}F_{n-3})+10$$



Where $k$ is a given number.
A friend of mine had this problem on a programming homework. And I wanted to impress him and told him there is a formula for this problem. But when I looked it more closely I realized it is not like other such problems that I have solved in discrete mathematics course, where we solved linear recurrence relations. So my question is could I find a formula for this recurrence relation and if I can how?










share|cite|improve this question











$endgroup$












  • $begingroup$
    @Did that's the Laplace Transform, right? My guess, the product $F_{n-2}times F_{n-3}$ spoils the fun.
    $endgroup$
    – tpb261
    Dec 18 '18 at 16:56












  • $begingroup$
    Hmmm... Somehow, I missed the product. Sorry.
    $endgroup$
    – Did
    Dec 18 '18 at 16:58






  • 2




    $begingroup$
    Doubt there is a simple formula here (as the growth is huge; and non-linear equations rarely have closed forms). As $ntoinfty$ we have $F_n sim kF_{n-2}F_{n-3}$ so if $F_n sim e^{g(n)}$ then $g(n) sim g(n-2) + g(n-3) + log(k)$ which has solutions on the form $g(n) sim r^n$ for some $r approx 1.32$ so asymptotically we will have $F_n sim e^{r^n}$ (this is all very rough).
    $endgroup$
    – Winther
    Dec 18 '18 at 17:18








  • 1




    $begingroup$
    One can probably prove that $lim_{ntoinfty}frac{log(log(F_n))}{n} = log(r)$ where $r$ is the real root of $r^3 - r - 1 = 0$
    $endgroup$
    – Winther
    Dec 18 '18 at 17:21






  • 1




    $begingroup$
    Linear recurrence problems necessarily have a closed form (if I am remembering correctly) and there are few others that we get lucky and we can solve as well... but generally speaking the non-linear cases won't have simple formulas. Just to echo what Winther said above...
    $endgroup$
    – Mason
    Dec 18 '18 at 20:50


















1












$begingroup$


I know how to solve linear homogeneous and non-homogeneous recurrence relations. But today I found this problem.



$$F_1=1$$
$$F_2=7$$
$$F_3=13$$
$$F_n = 3F_{n-1} + k(F_{n-2}F_{n-3})+10$$



Where $k$ is a given number.
A friend of mine had this problem on a programming homework. And I wanted to impress him and told him there is a formula for this problem. But when I looked it more closely I realized it is not like other such problems that I have solved in discrete mathematics course, where we solved linear recurrence relations. So my question is could I find a formula for this recurrence relation and if I can how?










share|cite|improve this question











$endgroup$












  • $begingroup$
    @Did that's the Laplace Transform, right? My guess, the product $F_{n-2}times F_{n-3}$ spoils the fun.
    $endgroup$
    – tpb261
    Dec 18 '18 at 16:56












  • $begingroup$
    Hmmm... Somehow, I missed the product. Sorry.
    $endgroup$
    – Did
    Dec 18 '18 at 16:58






  • 2




    $begingroup$
    Doubt there is a simple formula here (as the growth is huge; and non-linear equations rarely have closed forms). As $ntoinfty$ we have $F_n sim kF_{n-2}F_{n-3}$ so if $F_n sim e^{g(n)}$ then $g(n) sim g(n-2) + g(n-3) + log(k)$ which has solutions on the form $g(n) sim r^n$ for some $r approx 1.32$ so asymptotically we will have $F_n sim e^{r^n}$ (this is all very rough).
    $endgroup$
    – Winther
    Dec 18 '18 at 17:18








  • 1




    $begingroup$
    One can probably prove that $lim_{ntoinfty}frac{log(log(F_n))}{n} = log(r)$ where $r$ is the real root of $r^3 - r - 1 = 0$
    $endgroup$
    – Winther
    Dec 18 '18 at 17:21






  • 1




    $begingroup$
    Linear recurrence problems necessarily have a closed form (if I am remembering correctly) and there are few others that we get lucky and we can solve as well... but generally speaking the non-linear cases won't have simple formulas. Just to echo what Winther said above...
    $endgroup$
    – Mason
    Dec 18 '18 at 20:50
















1












1








1





$begingroup$


I know how to solve linear homogeneous and non-homogeneous recurrence relations. But today I found this problem.



$$F_1=1$$
$$F_2=7$$
$$F_3=13$$
$$F_n = 3F_{n-1} + k(F_{n-2}F_{n-3})+10$$



Where $k$ is a given number.
A friend of mine had this problem on a programming homework. And I wanted to impress him and told him there is a formula for this problem. But when I looked it more closely I realized it is not like other such problems that I have solved in discrete mathematics course, where we solved linear recurrence relations. So my question is could I find a formula for this recurrence relation and if I can how?










share|cite|improve this question











$endgroup$




I know how to solve linear homogeneous and non-homogeneous recurrence relations. But today I found this problem.



$$F_1=1$$
$$F_2=7$$
$$F_3=13$$
$$F_n = 3F_{n-1} + k(F_{n-2}F_{n-3})+10$$



Where $k$ is a given number.
A friend of mine had this problem on a programming homework. And I wanted to impress him and told him there is a formula for this problem. But when I looked it more closely I realized it is not like other such problems that I have solved in discrete mathematics course, where we solved linear recurrence relations. So my question is could I find a formula for this recurrence relation and if I can how?







discrete-mathematics recurrence-relations recursion






share|cite|improve this question















share|cite|improve this question













share|cite|improve this question




share|cite|improve this question








edited Dec 18 '18 at 17:02









rafa11111

1,1952417




1,1952417










asked Dec 18 '18 at 16:34









D.DimovD.Dimov

61




61












  • $begingroup$
    @Did that's the Laplace Transform, right? My guess, the product $F_{n-2}times F_{n-3}$ spoils the fun.
    $endgroup$
    – tpb261
    Dec 18 '18 at 16:56












  • $begingroup$
    Hmmm... Somehow, I missed the product. Sorry.
    $endgroup$
    – Did
    Dec 18 '18 at 16:58






  • 2




    $begingroup$
    Doubt there is a simple formula here (as the growth is huge; and non-linear equations rarely have closed forms). As $ntoinfty$ we have $F_n sim kF_{n-2}F_{n-3}$ so if $F_n sim e^{g(n)}$ then $g(n) sim g(n-2) + g(n-3) + log(k)$ which has solutions on the form $g(n) sim r^n$ for some $r approx 1.32$ so asymptotically we will have $F_n sim e^{r^n}$ (this is all very rough).
    $endgroup$
    – Winther
    Dec 18 '18 at 17:18








  • 1




    $begingroup$
    One can probably prove that $lim_{ntoinfty}frac{log(log(F_n))}{n} = log(r)$ where $r$ is the real root of $r^3 - r - 1 = 0$
    $endgroup$
    – Winther
    Dec 18 '18 at 17:21






  • 1




    $begingroup$
    Linear recurrence problems necessarily have a closed form (if I am remembering correctly) and there are few others that we get lucky and we can solve as well... but generally speaking the non-linear cases won't have simple formulas. Just to echo what Winther said above...
    $endgroup$
    – Mason
    Dec 18 '18 at 20:50




















  • $begingroup$
    @Did that's the Laplace Transform, right? My guess, the product $F_{n-2}times F_{n-3}$ spoils the fun.
    $endgroup$
    – tpb261
    Dec 18 '18 at 16:56












  • $begingroup$
    Hmmm... Somehow, I missed the product. Sorry.
    $endgroup$
    – Did
    Dec 18 '18 at 16:58






  • 2




    $begingroup$
    Doubt there is a simple formula here (as the growth is huge; and non-linear equations rarely have closed forms). As $ntoinfty$ we have $F_n sim kF_{n-2}F_{n-3}$ so if $F_n sim e^{g(n)}$ then $g(n) sim g(n-2) + g(n-3) + log(k)$ which has solutions on the form $g(n) sim r^n$ for some $r approx 1.32$ so asymptotically we will have $F_n sim e^{r^n}$ (this is all very rough).
    $endgroup$
    – Winther
    Dec 18 '18 at 17:18








  • 1




    $begingroup$
    One can probably prove that $lim_{ntoinfty}frac{log(log(F_n))}{n} = log(r)$ where $r$ is the real root of $r^3 - r - 1 = 0$
    $endgroup$
    – Winther
    Dec 18 '18 at 17:21






  • 1




    $begingroup$
    Linear recurrence problems necessarily have a closed form (if I am remembering correctly) and there are few others that we get lucky and we can solve as well... but generally speaking the non-linear cases won't have simple formulas. Just to echo what Winther said above...
    $endgroup$
    – Mason
    Dec 18 '18 at 20:50


















$begingroup$
@Did that's the Laplace Transform, right? My guess, the product $F_{n-2}times F_{n-3}$ spoils the fun.
$endgroup$
– tpb261
Dec 18 '18 at 16:56






$begingroup$
@Did that's the Laplace Transform, right? My guess, the product $F_{n-2}times F_{n-3}$ spoils the fun.
$endgroup$
– tpb261
Dec 18 '18 at 16:56














$begingroup$
Hmmm... Somehow, I missed the product. Sorry.
$endgroup$
– Did
Dec 18 '18 at 16:58




$begingroup$
Hmmm... Somehow, I missed the product. Sorry.
$endgroup$
– Did
Dec 18 '18 at 16:58




2




2




$begingroup$
Doubt there is a simple formula here (as the growth is huge; and non-linear equations rarely have closed forms). As $ntoinfty$ we have $F_n sim kF_{n-2}F_{n-3}$ so if $F_n sim e^{g(n)}$ then $g(n) sim g(n-2) + g(n-3) + log(k)$ which has solutions on the form $g(n) sim r^n$ for some $r approx 1.32$ so asymptotically we will have $F_n sim e^{r^n}$ (this is all very rough).
$endgroup$
– Winther
Dec 18 '18 at 17:18






$begingroup$
Doubt there is a simple formula here (as the growth is huge; and non-linear equations rarely have closed forms). As $ntoinfty$ we have $F_n sim kF_{n-2}F_{n-3}$ so if $F_n sim e^{g(n)}$ then $g(n) sim g(n-2) + g(n-3) + log(k)$ which has solutions on the form $g(n) sim r^n$ for some $r approx 1.32$ so asymptotically we will have $F_n sim e^{r^n}$ (this is all very rough).
$endgroup$
– Winther
Dec 18 '18 at 17:18






1




1




$begingroup$
One can probably prove that $lim_{ntoinfty}frac{log(log(F_n))}{n} = log(r)$ where $r$ is the real root of $r^3 - r - 1 = 0$
$endgroup$
– Winther
Dec 18 '18 at 17:21




$begingroup$
One can probably prove that $lim_{ntoinfty}frac{log(log(F_n))}{n} = log(r)$ where $r$ is the real root of $r^3 - r - 1 = 0$
$endgroup$
– Winther
Dec 18 '18 at 17:21




1




1




$begingroup$
Linear recurrence problems necessarily have a closed form (if I am remembering correctly) and there are few others that we get lucky and we can solve as well... but generally speaking the non-linear cases won't have simple formulas. Just to echo what Winther said above...
$endgroup$
– Mason
Dec 18 '18 at 20:50






$begingroup$
Linear recurrence problems necessarily have a closed form (if I am remembering correctly) and there are few others that we get lucky and we can solve as well... but generally speaking the non-linear cases won't have simple formulas. Just to echo what Winther said above...
$endgroup$
– Mason
Dec 18 '18 at 20:50












0






active

oldest

votes











Your Answer





StackExchange.ifUsing("editor", function () {
return StackExchange.using("mathjaxEditing", function () {
StackExchange.MarkdownEditor.creationCallbacks.add(function (editor, postfix) {
StackExchange.mathjaxEditing.prepareWmdForMathJax(editor, postfix, [["$", "$"], ["\\(","\\)"]]);
});
});
}, "mathjax-editing");

StackExchange.ready(function() {
var channelOptions = {
tags: "".split(" "),
id: "69"
};
initTagRenderer("".split(" "), "".split(" "), channelOptions);

StackExchange.using("externalEditor", function() {
// Have to fire editor after snippets, if snippets enabled
if (StackExchange.settings.snippets.snippetsEnabled) {
StackExchange.using("snippets", function() {
createEditor();
});
}
else {
createEditor();
}
});

function createEditor() {
StackExchange.prepareEditor({
heartbeatType: 'answer',
autoActivateHeartbeat: false,
convertImagesToLinks: true,
noModals: true,
showLowRepImageUploadWarning: true,
reputationToPostImages: 10,
bindNavPrevention: true,
postfix: "",
imageUploader: {
brandingHtml: "Powered by u003ca class="icon-imgur-white" href="https://imgur.com/"u003eu003c/au003e",
contentPolicyHtml: "User contributions licensed under u003ca href="https://creativecommons.org/licenses/by-sa/3.0/"u003ecc by-sa 3.0 with attribution requiredu003c/au003e u003ca href="https://stackoverflow.com/legal/content-policy"u003e(content policy)u003c/au003e",
allowUrls: true
},
noCode: true, onDemand: true,
discardSelector: ".discard-answer"
,immediatelyShowMarkdownHelp:true
});


}
});














draft saved

draft discarded


















StackExchange.ready(
function () {
StackExchange.openid.initPostLogin('.new-post-login', 'https%3a%2f%2fmath.stackexchange.com%2fquestions%2f3045372%2fhow-to-solve-this-nonlinear-recurrence-relations%23new-answer', 'question_page');
}
);

Post as a guest















Required, but never shown

























0






active

oldest

votes








0






active

oldest

votes









active

oldest

votes






active

oldest

votes
















draft saved

draft discarded




















































Thanks for contributing an answer to Mathematics Stack Exchange!


  • Please be sure to answer the question. Provide details and share your research!

But avoid



  • Asking for help, clarification, or responding to other answers.

  • Making statements based on opinion; back them up with references or personal experience.


Use MathJax to format equations. MathJax reference.


To learn more, see our tips on writing great answers.




draft saved


draft discarded














StackExchange.ready(
function () {
StackExchange.openid.initPostLogin('.new-post-login', 'https%3a%2f%2fmath.stackexchange.com%2fquestions%2f3045372%2fhow-to-solve-this-nonlinear-recurrence-relations%23new-answer', 'question_page');
}
);

Post as a guest















Required, but never shown





















































Required, but never shown














Required, but never shown












Required, but never shown







Required, but never shown

































Required, but never shown














Required, but never shown












Required, but never shown







Required, but never shown







Popular posts from this blog

Le Mesnil-Réaume

Ida-Boy-Ed-Garten

web3.py web3.isConnected() returns false always